How to Verify Green's Theorem for a Given Rectangle?

Click For Summary
Green's Theorem can be verified for the given rectangle by converting the line integral into a double integral and solving it, which yields a result that needs confirmation. The discussion highlights two methods for evaluating the line integral: parameterizing each segment of the rectangle or directly integrating along the specified paths without parameterization. The parameterization approach involves breaking down the path into four segments and calculating the integrals for each segment. Alternatively, the direct method simplifies the process by integrating along the defined vertices of the rectangle. Both methods should yield consistent results to confirm the validity of Green's Theorem.
sam121
Messages
1
Reaction score
0

Homework Statement



Verify Green's Theorem in the plane for the \oint [(x^{2} - xy^{2})dx + (y^{3} + 2xy)dy] where C is a rectangle with vertices at (-1,-2), (1,-2), (1,1) and (-1,1).


The Attempt at a Solution



This means you have to use green's theorem to convert it into a double integral and solve which I have done. Not 100% sure whether the answer is 6 or 12 however. You then have to do the line integral directly to verify you get the same answer. This is where I get stuck. Please could I have some help on how to do this line integral directly? thank you :)
 
Physics news on Phys.org
You can do it a couple different ways:

One is to parameterize each segment of the rectangle and evaluate four integrals in terms of t. For example, the path from (-1, -2) to (1, -2) can be parameterized as \stackrel{\rightarrow}{r}(t) = \left\langle -1 + 2t, -2\right\rangle where 0 \leq t \leq 1, so you can have x = -1 + 2t, y = -2, dx = 2dt, and dy = 0dt. Then just substitute everything and evaluate, then repeat the process three more times.

Another way, since all the paths are straight lines either vertical or horizontal, is to evaluate the integrals without parametizing the path. Make a specific path of vertices, say the path you listed of (-1,-2) \rightarrow (1,-2) \rightarrow (1,1) \rightarrow (-1,1) \rightarrow (-1, -2), then create integrals from each of those paths. For example, for the first path x goes from -1 to 1 and y = -2 and does not change, so the integral becomes:
\int^{1}_{-1}x^{2} - 4xdx.

Either way will work.
 
Question: A clock's minute hand has length 4 and its hour hand has length 3. What is the distance between the tips at the moment when it is increasing most rapidly?(Putnam Exam Question) Answer: Making assumption that both the hands moves at constant angular velocities, the answer is ## \sqrt{7} .## But don't you think this assumption is somewhat doubtful and wrong?

Similar threads

  • · Replies 10 ·
Replies
10
Views
2K
Replies
3
Views
2K
  • · Replies 4 ·
Replies
4
Views
2K
  • · Replies 3 ·
Replies
3
Views
2K
  • · Replies 1 ·
Replies
1
Views
2K
Replies
12
Views
2K
  • · Replies 2 ·
Replies
2
Views
2K
  • · Replies 6 ·
Replies
6
Views
3K
  • · Replies 2 ·
Replies
2
Views
2K
  • · Replies 2 ·
Replies
2
Views
2K